আমরা কি (তাত্ত্বিকভাবে) ব্ল্যাকহোলকে এত শক্ত করে স্পিন করতে পারি যে সেন্ট্রিফুগাল বল দ্বারা এটি ভেঙে যাবে?


26

আমি ব্ল্যাকহোলের জীবনে জড়িত শক্তিগুলি কল্পনা করতে পারি না। সুতরাং দয়া করে, এই সুনির্দিষ্ট উপায়ে ব্ল্যাকহোলকে ধ্বংস করা সম্ভব কিনা তা জানতে আমাকে সহায়তা করুন।


আমি নিশ্চিত নই, তবে আমি মনে করি যে এটির স্পিন দক্ষতার সাথে বাড়ানোর জন্য আপনাকে এটিকে স্টাফ নিক্ষেপ করতে হবে এবং যদি আপনি এটি করেন তবে আপনি এর ভরও বাড়িয়ে তুলবেন; জিআর এর শক্তিশালী ক্ষেত্রের সীমাতে, সংশ্লিষ্ট গণ বৃদ্ধি কেন্দ্রীভূত শক্তিটিকে এটি পৃথকীকরণ থেকে বাধা দেয়। সর্বোপরি এটি একটি কেরার ব্ল্যাকহোল হয়ে যায় যার সাথে 'এ' তাত্পর্যপূর্ণভাবে 1 টির কাছাকাছি থাকে। ব্ল্যাক হোলের জন্য, মহাকর্ষ সর্বদা জয়ী হয়! (যদি আপনি জিনিস ফেলে না দিয়ে এটিকে টর্চ করতে চান তবে আপনাকে বেশ দূরে থাকতে হবে এবং এটি খুব দক্ষ হবে না)
ক্রিস

উত্তর:


23

আমরা (তাত্ত্বিকভাবে) ব্ল্যাকহোলকে এত শক্ত করে স্পিন করতে পারি যাতে কেন্দ্রীভূত শক্তি দ্বারা এটি ভেঙে যায়?

একটি কেরার-নিউম্যানের (ঘূর্ণন, চার্জড, বিচ্ছিন্ন) ভর , ব্ল্যাকহোল , কৌণিক গতিবেগ এবং চার্জ কিয়ের জন্য , ইভেন্ট দিগন্তের পৃষ্ঠের ক্ষেত্রফল A = 8 M দ্বারা দেওয়া হয়েছে [ এম 2 + ( এম 2 - একটি 2) - প্রশ্নঃ 2 ) 1 / 2 - প্রশ্নঃ 2 / 2 ] , যেখানে একটি = জে / এম । একটি একস্ট্রিমাল কালো গহ্বর ঘটে যখন এম 2 = একটিএমজেপ্রশ্নঃ

একজন=8এম[এম2+ +(এম2-একটি2-প্রশ্নঃ2)1/2-প্রশ্নঃ2/2],
একটি=জে/এম । এর বাইরে, যদি ব্ল্যাকহোলটি আরও বেশি ওভারস্পান বা অতিরিক্ত চার্জযুক্ত হয় তবে এটি একটি "ওভারটেক্সট্রিমাল" কের-নিউম্যান স্পেসটাইম, যা আসলেই কোনও ব্ল্যাক হোল হবে না, বরং একটি নগ্ন একাকীত্ব।এম2=একটি2+ +প্রশ্নঃ2

সুতরাং, আমি আপনার প্রশ্নের ব্যাখ্যা হিসাবে জিজ্ঞাসা করি যে ব্ল্যাকহোলটি অতিমাত্রার সীমা এবং তার বাইরেও ছড়িয়ে দেওয়া যেতে পারে, যাতে ইভেন্টের দিগন্তকে ধ্বংস করা যায়। এটি খুব সম্ভব যে এটি করা যায় না।

১৯d৪ সালে ওয়াল্ড প্রমাণ করেছেন যে কেউ যখন তার কৌনিক গতি বাড়ানোর চেষ্টা করার জন্য একটি কৃষ্ণগহ্বরের মধ্যে ঝাপিয়ে পড়েছে, এটি একটি অতিমাত্রার ব্ল্যাক হোলের নিকটতম, এই প্রক্রিয়াটি চালিয়ে নেওয়া তত বেশি কঠিন: একটি দ্রুত স্পিনিং ব্ল্যাকহোল বিষয়টি দূরে রাখবে এটি অত্যন্ত সীমা ছাড়িয়ে যাবে। অন্যান্য স্কীম রয়েছে, এবং যদিও আমি শাস্ত্রীয় সাধারণ আপেক্ষিকতার মধ্যে সম্পূর্ণ সাধারণ প্রমাণ সম্পর্কে অবগত নই , ব্ল্যাকহোল ডায়নামিক্স এবং থার্মোডাইনামিক্সের সংযোগের মাধ্যমে এ জাতীয় প্রকল্পগুলির ক্রমাগত ব্যর্থতা ভালভাবে অনুপ্রাণিত হয়।

টিএইচ=κ/2π

κ=এম2-একটি2-প্রশ্নঃ22এম(এম+ +এম2-একটি2-প্রশ্নঃ2)-প্রশ্নঃ2

2

আমার মাথার উপরের অংশ থেকে সমস্ত গণিত নেই, তবে আমার ধারণাগত বোঝা থেকে এটি সম্ভব নয়।

কৃষ্ণ গহ্বরগুলিতে একটি বৃহত পর্যায়ে মহাকর্ষীয় আকর্ষণ রয়েছে যা এমনকি আলোক "পৃষ্ঠ" এর বাইরেও ভালভাবে বাঁচতে পারে না (এটি যদি ব্ল্যাকহোলের পর্যাপ্ত পরিমাণ থাকে তবে এটির পৃষ্ঠ এখনও থাকে এবং এককতার মধ্যে পড়ে না)। এর অর্থ হ'ল এটি যথেষ্ট দ্রুত স্পিন করতে হবে যে পর্যাপ্ত রৈখিক গতিবেগ (প্রায়শই কথোপকথনকে একটি বৃত্তাকার রেফারেন্স ফ্রেমে "সেন্ট্রিফুগাল ফোর্স বলা হয়)" রক্ষা পাওয়ার জন্য পৃষ্ঠটি আলোর গতির চেয়ে উল্লেখযোগ্যভাবে দ্রুত গতিতে চলেছে, যা অনুসারে আপেক্ষিকতা তত্ত্ব সম্ভব নয়।

হকিং বিকিরণ কেবল তখনই সম্ভব কারণ বৈদ্যুতিন চৌম্বকীয় বিকিরণটি ব্লাকহোলের "পৃষ্ঠ" এর দিকে orthogonally খুব কাছাকাছি চলেছে এবং আলো কেবল মাধ্যাকর্ষণ দ্বারা "বাঁকানো" হতে পারে, এটি একটি স্টপে টানা যায় না।


2
এই উত্তরটি নিয়ে বেশ কয়েকটি ধারণামূলক সমস্যা রয়েছে। এটি সুপারিশ করে যে "পৃষ্ঠতল" ঘটনা দিগন্তের বাইরে অন্য কিছু - তবে এটি কী? এটি পরামর্শ দেয় যে হকিং বিকিরণ তড়িৎ চৌম্বকীয় জাতীয় ভর বিহীন বিকিরণের মধ্যে সীমাবদ্ধ - সত্য নয়। এটি সুপারিশ করে যে ব্ল্যাক হোলগুলি আলো থামাতে পারে না - দিগন্তটি একটি হালকা ধরণের পৃষ্ঠ যা এটি করে।
স্ট্যান লিউ


0

আমরা যতদূর জানি ব্ল্যাকহোল থামাতে পারে এমন কিছুই নেই। এই ধারণাটি বোঝার জন্য আপনাকে প্রথমে ব্ল্যাক হোল সম্পর্কে যা জানা আছে তা প্রথমে দেখতে হবে । একবার আপনি এটি উপলব্ধি করতে পারবেন, তবে আপনি দেখতে পাবেন যে কসমস সম্পর্কে আমাদের বর্তমান বোঝার কারণে আমরা কৃষ্ণগহ্বরের কিছুই করতে পারি না।

এটি সত্য যে হকিং রেডিয়েশন কোনও ব্ল্যাকহোলকে প্রভাবিত করতে পারে তবে এটি কেবল খুব খুব ছোট ব্ল্যাক হোলের জন্য।

যাইহোক, পদার্থবিজ্ঞানে কোনও কেন্দ্রীভূত শক্তি নেই - এটি আসলে অনেকেরই একটি ভুল ধারণা। তবে সেন্ট্রিপেটাল বল রয়েছে

এখানে চিত্র বর্ণনা লিখুন


2
বাধ্যতামূলক প্রত্যাখ্যান-মাধ্যমে-
এক্সকেসিডি

3
@ ইলমারিকারোনেন নিচের ভোটের জন্য ধন্যবাদ, তবে আপনি কি বুঝতে পেরেছেন যে কার্টুনটি সঠিক কিছু প্রমাণ করে না? সেন্ট্রিপেটাল বল এবং কেন্দ্রকেন্দ্রের মধ্যে পার্থক্য রয়েছে।
FunctionR

2
মোটামুটি, কার্টুনটি নির্দেশ করে যে সেন্ট্রিফুগল শক্তি যেমন জড় শক্তি, তেমনি সত্য। যুক্তিযুক্তভাবে আন্ত: বাহিনীকে "আসল নয়" হিসাবে ব্যাখ্যা করা বুদ্ধিমানের, তবে আমি নিশ্চিত নই যে এই প্রশ্নটির কী প্রভাব রয়েছে, কেননা মহাকর্ষীয় শক্তিও একটি জড় শক্তি।
স্টান লিউ

এই "উত্তর "টি বেশ অস্পষ্ট, প্রশ্নবিদ্ধ বিবৃতি দেয় এবং মূল ক্যোয়ারীর কোনও সত্যই জন্মায় না।
ফ্লোরিন আন্দ্রেই

0

মজাদার. এই প্রক্রিয়াটি প্রথম স্থানে ব্ল্যাকহোল গঠনে প্রভাব ফেলতে পারে। একটি ঘূর্ণনকারী তারা বিবেচনা করুন যা মহাকর্ষীয় শক্তির কারণে মারা যায় এবং সঙ্কুচিত হতে শুরু করে। এটি সঙ্কুচিত হওয়ার সাথে সাথে এর সমস্ত ভর আরও একটি ছোট ব্যাসার্ধে কমপ্যাক্ট হবে। এর দুটি পরিণতি হবে: ১) শরীরের বিভিন্ন অংশে আকর্ষণের মাধ্যাকর্ষণ শক্তি স্কোয়ারাস ব্যাসার্ধের বিপরীত দিয়ে বেড়ে উঠবে এবং ২) এর ঘূর্ণন গতি কৌণিক গতিবেগ সংরক্ষণ এবং প্রসারিত বলের কারণে বৃদ্ধি পাবে, ব্যাসার্ধ ঘনক্ষেত্রের বিপরীতে বড় হবে। এর অর্থ হ'ল চুক্তি করার চেয়ে সম্প্রসারণ শক্তি আরও দ্রুত বৃদ্ধি পাবে এবং কমপক্ষে নিউটনীয় দৃষ্টিতে প্রসারিত শক্তি জিতবে। এই দৃষ্টিকোণ থেকে, দেখে মনে হচ্ছে একটি ঘূর্ণায়মান তারা কখনই ব্ল্যাকহোল তৈরি করে না ...


আপনি এই নিবন্ধটি পছন্দ করতে পারেন। slate.com/blogs/quora/2014/02/10/…
userLTK

-1

আসুন নিম্নলিখিত চেষ্টা করুন:

বাহিনী সমান:

এফ=মিবনাম2আরএফ=জিএমমিআর2মিবনাম2/আর=জিএমমিআর2বনাম2=জিএমআর

ব্যাসার্ধটি যেহেতু আরগুলি=2জিএম/2

বনাম2=জিএম2জিএম/2বনাম=2/2বনাম=2বনাম=0,707

আমি জানি না আমি সমস্ত বিবেচনা করেছি কিনা, তবে আমার জ্ঞান যতদূর যায়, যদি ব্ল্যাকহোলটি তার চেয়ে দ্রুত গতিতে চলেছে 0,707, ইভেন্ট দিগন্ত বর্তমান ব্যাসার্ধে নিজেকে বজায় রাখতে সক্ষম হবে না।

যাইহোক, যখন ব্যাসার্ধটি প্রসারিত হবে তখন কৌণিক গতি সংরক্ষণের মাধ্যমে আবর্তনটি ধীর হয়ে যাবে ... সুতরাং আমি মনে করি না এটি ছিঁড়ে যাবে ... সম্ভবত "ধূসর গর্ত" হয়ে যাবে?

যদি কোনও মৌলিক ভুল হয় তবে আমাকে ক্ষমা করুন, আমি এই সবের জন্য নতুন ...: পি


3
এটাকে নিউটনিয়ান দেখাচ্ছে। আপনি কি একটি ব্ল্যাক হোলের কাছাকাছি স্পেসটাইমের বক্রতা বিবেচনা করেছেন? ব্ল্যাক হোলের জন্য "কের সমাধান "ও দেখুন See
জেরাল্ড

আমি "গ্রে হোল" শব্দটি পছন্দ করি।
ইউজারএলটিকে

এটি একটি পুরানো উত্তর, তবে আমি ভেবেছিলাম আমি এটি এখানে ফেলে দেব কারণ এটি ব্ল্যাক হোলের কক্ষপথের বেগকে সম্বোধন করে। আপনার পালানোর জন্য ব্ল্যাকহোলের তাত্ত্বিক প্রান্তটি ইভেন্টের দিগন্ত সি তে স্পিনিং করা দরকার যা স্পষ্টতই অসম্ভব। physics.stackexchange.com/questions/207816/...
userLTK
আমাদের সাইট ব্যবহার করে, আপনি স্বীকার করেছেন যে আপনি আমাদের কুকি নীতি এবং গোপনীয়তা নীতিটি পড়েছেন এবং বুঝতে পেরেছেন ।
Licensed under cc by-sa 3.0 with attribution required.